Integration with constant of proportionality

Click For Summary
The discussion revolves around solving a differential equation involving the constant of proportionality, k. The user initially presents their solution method, integrating each term separately, but questions its validity compared to the solution manual's approach. It is clarified that both methods are valid, but an integration constant must be included in the final answer. The user acknowledges the oversight regarding the integration constant after receiving feedback. The conversation emphasizes the importance of correctly handling constants during integration.
GeoMike
Messages
64
Reaction score
0
This seems easy enough, but now I'm second guessing myself...

Problem:
http://www.mcschell.com/probl.gif


I solved it as follows:
<br /> \frac{dN}{ds} = k(250-s)
dN = k(250-s) ds
\int dN = \int k(250-s) ds
N = k\int (250-s) ds
N = k(\int 250 ds - \int s ds)
N = k(250s - \frac{s^2}{2}) + C

But this is what the solution manual gives:
http://www.mcschell.com/solu.gif

I understand how they worked the problem, (substituting with u = (250 - s) and then integrating), but I didn't think this was the correct way to handle this problem. Why can't you just integrate each term seperately after pulling out the constant k[\b]?
My reasoining was that if you distributed the k before integrating you'd end up with my anser, not the solution guide's...

Thanks,
-GeoMike-
 
Last edited by a moderator:
Physics news on Phys.org
You can do it both ways. Your way is equivalent to the answer in the textbook (however, you need an integration constant!)

If you expand the answer in the book, you will get k(250s-s2/2+2502/2) +C. However, note that the last term in the bracket is simply a constant, and so your integration constant (D, say) will be equal to C +(2502k)/2.
 
Last edited:
cristo said:
(however, you need an integration constant!)

ACK! My mistake! Thanks!

-GeoMike-
 
Question: A clock's minute hand has length 4 and its hour hand has length 3. What is the distance between the tips at the moment when it is increasing most rapidly?(Putnam Exam Question) Answer: Making assumption that both the hands moves at constant angular velocities, the answer is ## \sqrt{7} .## But don't you think this assumption is somewhat doubtful and wrong?

Similar threads

Replies
2
Views
1K
Replies
6
Views
2K
  • · Replies 1 ·
Replies
1
Views
1K
  • · Replies 21 ·
Replies
21
Views
2K
  • · Replies 1 ·
Replies
1
Views
1K
  • · Replies 2 ·
Replies
2
Views
1K
  • · Replies 6 ·
Replies
6
Views
3K
Replies
3
Views
2K
  • · Replies 4 ·
Replies
4
Views
2K
  • · Replies 1 ·
Replies
1
Views
2K